- PowerScore Staff
- Posts: 5972
- Joined: Mar 25, 2011
- Fri Jan 21, 2011 12:00 am
#41279
Complete Question Explanation
(The complete setup for this game can be found here: lsat/viewtopic.php?t=11660)
The correct answer choice is (D)
Note that the question stem asks for a complete and accurate list, and the correct answer must list all of the factories that could appear on day 1. So, the correct answer must contain each and every factory that is possible for day 1. As strange as it may sound, the question asks for what must be true about what is possible.
If G and H are to be scheduled as far apart as possible (that is, the distance between them is maximized), this would likely place G and H in days 1 and 5 or days 2 and 6 (do not forget that F must be in either day 1 or day 6). Again, there are several ways to approach this question depending on your level of game understanding. At the most advanced level, answer choices (A), (C), and (E) can likely be eliminated since each contains J. If J is inspected on day 1, then F would be inspected on day 6 and G and H would not be as far apart as possible—a violation of the “if” clause in the question. Only F differentiates answer choice (B) from answer choice (D), and since F is a day 1 or day 6 player, it seems likely that answer choice (D) is correct, and in fact it is the correct answer. If that type of theoretical analysis makes you a bit nervous, you can always resort to the other form of attack on this question: make a few quick hypotheticals. Here is one that eliminates answer choices (B) and (C): F-G-J-Q-R-H. Another that eliminates answer choice (A) is: G-J-Q-R-H-F. And again, the inclusion of J in answer choice (E) should be a tip-off that answer choice (E) is likely incorrect.
(The complete setup for this game can be found here: lsat/viewtopic.php?t=11660)
The correct answer choice is (D)
Note that the question stem asks for a complete and accurate list, and the correct answer must list all of the factories that could appear on day 1. So, the correct answer must contain each and every factory that is possible for day 1. As strange as it may sound, the question asks for what must be true about what is possible.
If G and H are to be scheduled as far apart as possible (that is, the distance between them is maximized), this would likely place G and H in days 1 and 5 or days 2 and 6 (do not forget that F must be in either day 1 or day 6). Again, there are several ways to approach this question depending on your level of game understanding. At the most advanced level, answer choices (A), (C), and (E) can likely be eliminated since each contains J. If J is inspected on day 1, then F would be inspected on day 6 and G and H would not be as far apart as possible—a violation of the “if” clause in the question. Only F differentiates answer choice (B) from answer choice (D), and since F is a day 1 or day 6 player, it seems likely that answer choice (D) is correct, and in fact it is the correct answer. If that type of theoretical analysis makes you a bit nervous, you can always resort to the other form of attack on this question: make a few quick hypotheticals. Here is one that eliminates answer choices (B) and (C): F-G-J-Q-R-H. Another that eliminates answer choice (A) is: G-J-Q-R-H-F. And again, the inclusion of J in answer choice (E) should be a tip-off that answer choice (E) is likely incorrect.
Dave Killoran
PowerScore Test Preparation
Follow me on X/Twitter at http://twitter.com/DaveKilloran
My LSAT Articles: http://blog.powerscore.com/lsat/author/dave-killoran
PowerScore Podcast: http://www.powerscore.com/lsat/podcast/
PowerScore Test Preparation
Follow me on X/Twitter at http://twitter.com/DaveKilloran
My LSAT Articles: http://blog.powerscore.com/lsat/author/dave-killoran
PowerScore Podcast: http://www.powerscore.com/lsat/podcast/